Mecánica clásica de la mecánica cuántica

Estoy buscando una manera de probar que uno recupera, bajo suposiciones ad hoc, la mecánica clásica de la teoría cuántica. Por lo general, podemos encontrar en los libros de texto que el propagador

k ( X , X 0 ; t ) = X | mi i H ^ t / | X 0

se da en el límite clásico usando la formulación de la integral de trayectoria

k ( X , X 0 ; t ) mi i S [ q C ( t ) ]

dónde S [ q ] es la acción clásica y q C ( t ) es el camino clásico de X 0 a X en un tiempo t (que minimiza S ).

Pero esto no es muy satisfactorio, ya que k es una amplitud de probabilidad, que en realidad no es lo que nos gustaría en el límite clásico. Lo mínimo que nos gustaría obtener es que la probabilidad de estar en X en el momento t es dado por

(1) PAG t ( X ) = d ( X q C ( t ) )

o algo equivalente. Mi enfoque ingenuo para recuperar este resultado sería el siguiente. El propagador no es suficiente, ya que el "clasicismo" del movimiento debería estar dado por el estado inicial. Parece natural elegir que la función de onda inicial sea un paquete de onda con un pico alrededor de la posición inicial (en x = 0 a partir de ahora) con dispersión Δ X y con impulso pag , Por ejemplo

ψ ( X ) = mi X 2 2 Δ X 2 i pag X / π 1 / 4 Δ X

Diríamos que la dinámica será clásica al menos si Δ pag / Δ X pag . Tal vez haya otras limitaciones, por ejemplo, la dinámica es clásica solo a veces. t el tiempo suficiente, pero esto todavía no está claro para mí.

Finalmente, la probabilidad clásica viene dada por (o al menos proporcional a)

(2) PAG t ( X ) = | d X 0 k ( X , X 0 ; t ) ψ ( X 0 ) | 2 | d X 0 mi i S [ q C ( t ) ] ψ ( X 0 ) | 2 .

Mi pregunta es: ¿hay alguna manera de mostrar/probar que, bajo estos supuestos, podemos obtener (1) de (2) para cualquier hamiltoniano? He mirado el caso más simple de una partícula libre, y parece funcionar (todavía tengo algunos problemas para obtener el resultado final, pero mi sensación es que funciona). Si ayuda, podría publicar el cálculo más tarde. Pero una prueba general sería genial.

Quizás te interese leer esto motls.blogspot.com/2011/11/… Donde Lubos muestra cómo a partir de un conjunto de fotones se obtiene la onda electromagnética clásica.
@Qmechanic: había leído estas publicaciones, pero no creo que respondan exactamente a mi pregunta, que puede reformularse de una manera más general (sin referencia a las integrales de ruta): ¿Es posible demostrar que | ψ ( X , t ) | da a, bajo algunas condiciones iniciales particulares, una probabilidad de la forma de la ecuación (1). Pero tal vez pueda usar estas publicaciones para responder a mi pregunta.
@dj_mummy: Gracias por la referencia, es útil, aunque no aborda directamente mis preocupaciones (no quiero trabajar con los valores esperados, sino con la función de onda y la regla de Born).
No creo que (1) sea válido para todos los tiempos: el requisito para un tratamiento semiclásico es esencialmente que la acción sea pequeña en comparación con . Pero la acción de un camino dado depende del intervalo de tiempo de este camino y, por lo tanto, se debe restringir la atención a la t 0 límite. (La imagen clásica se ve borrosa por tiempos más largos). Por cierto, para que el enfoque semiclásico suene matemático, la aproximación WKB es el método preferido y no algunos argumentos heurísticos con integrales de ruta de Faymann.

Respuestas (1)

La cuestión de cómo la mecánica cuántica se reduce a la mecánica clásica en el límite 0 se ha preguntado varias veces antes, ver. por ejemplo , este , este y este Phys.SE publicaciones y enlaces en el mismo.

Yo dejo Δ t := t F t i y Δ X := X F X i . El hecho clave ahora es que el propagador /kernel/amplitude de Feynman k ( X F , t F ; X i , t i ) se localiza en una función delta

(A) k ( X F , t F ; X i , t i )     d ( Δ X ) para Δ t 0 + .

La distribución delta (A) tiene apoyo en

(B) X F   =   X i .

Heurísticamente, la ec. (A) sigue porque para tiempos suficientemente cortos | Δ t | τ , dónde τ es una escala de tiempo característica, el potencial V no tiene (por así decirlo) tiempo para interactuar, y el término cinético explota, por lo que se puede usar la fórmula gaussiana para una partícula libre.

II) Si no hay potencial V en el modelo, es decir, el modelo es una partícula libre, es suficiente asumir | Δ t | I , para derivar el límite (A), donde I es una escala característica (con dimensión igual a un momento de inercia), por lo que en este caso se puede considerar el límite 0 sin ir a tiempos cortos.

III) De manera más general, se pueden aplicar los métodos semiclásicos de WKB. En el caso de un oscilador armónico, la amplitud se vuelve

(C) k ( X F , t F ; X i , t i )     d ( ( X F 2 + X i 2 ) C 2 X F X i )   =   d ( ( X F 1 + s C X i ) ( X F 1 s C X i ) C ) para 0 + ,

dónde C := porque ω Δ t y s := pecado ω Δ t . La distribución delta (C) tiene soporte en

(D) X F = 1 ± s C X i .

IV) La condición (B) para la partícula libre y (D) para el oscilador armónico solo tiene sentido clásico propio en el límite de tiempo corto | Δ t | τ . Para grandes tiempos | Δ t | , el límite clásico (B) y (D) son un remanente del procedimiento de promedio cuántico en muchas historias, que no son del todo clásicas en el límite 0 . Una interpretación heurística es posiblemente una especie de estado mixto clásico.

V) La probabilidad

(MI) PAG ( X F , t F ; X i , t i )   =   | k ( X F , t F ; X i , t i ) | 2

en el límite (A) de un cuadrado de la distribución delta de Dirac está matemáticamente mal definido, cf. por ejemplo , este y este Phys.SE publicaciones. Con respecto a la normalización del propagador de Feynman y la interpretación como probabilidad, consulte también esta publicación de Phys.SE.